Invertible and Non-Invertible Systems (Solved Problems)

Поделиться
HTML-код
  • Опубликовано: 23 окт 2024
  • Signal and System: Solved Questions on Invertible and Non-Invertible Systems.
    Topics Discussed:
    1. Solved examples on invertible and non-invertible systems.
    2. Use of impulse and step signal as the input to find out if the system is invertible or non-invertible.
    3. Homework problems on invertible and non-invertible systems.
    Follow Neso Academy on Instagram: @nesoacademy(bit.ly/2XP63OE)
    Follow me on Instagram: @sujeetsingh20(bit.ly/2JLcQz5)
    Contribute: www.nesoacademy...
    Memberships: bit.ly/2U7YSPI
    Books: www.nesoacademy...
    Website ► www.nesoacademy...
    Forum ► forum.nesoacad...
    Facebook ► goo.gl/Nt0PmB
    Twitter ► / nesoacademy
    Music:
    Axol x Alex Skrindo - You [NCS Release]
    #SignalAndSystemByNeso #Signal&System #InvertibleSystem #NonInvertibleSystem

Комментарии • 182

  • @harikotha7
    @harikotha7 7 лет назад +82

    Neso Academy u are the best
    I've prepared for my interview 2 days back using ur digital electronics tutorials 👏👌
    can u start a new series about Microprocessors Microcontrollers and embedded systems

  • @jahansaid6382
    @jahansaid6382 5 лет назад +41

    I believe there are both non avertible; many to one maping. for the first function you plug -1 and +1 you will get the same answerr which is one. for the second question, when you take the derivative of any constant it could be always zero.

  • @samuelvargas4109
    @samuelvargas4109 6 лет назад +18

    Hello. When proving that a system is invertible, when should we use the step function instead of the impulse function?

  • @kirankumar2432
    @kirankumar2432 6 лет назад +8

    Sir,can we make a generalization here......that the systems which produce the output as the scaled version of input is always invertible...If not give me an example..@nesoacademy

  • @kevinjad4506
    @kevinjad4506 6 лет назад +33

    Sir Thank you so much.
    I couldn't believe that I'm getting a world class tutoring for free.
    It would be nice if you do a series on microcontrollers microprocessors and embedded systems.
    Since you have given us a solid background in digital electronics

    • @ritikroshan9681
      @ritikroshan9681 3 года назад +1

      Did you find some good resource of them plz reply

  • @dimitrispanapakidis3990
    @dimitrispanapakidis3990 Год назад +23

    HW PROBLEMS:
    1. Non-Invertible
    2. DEPENDS on wether dx(t)/dt is STRICTLY MONOTONIC or not
    EXPLANATION OF 2: dx(t)/dt is the slope of the function x(t). If this slope is STRICTLY MONOTONIC (i.e. if it is constantly increasing in value OR if it is constantly decreasing in value), then it follows 1 to 1 mapping ("1-1"). And thus it is INVERTIBLE.
    Otherwise, it is Non-Invertible.
    * How to find if dx(t)/dt is STRICTLY MONOTONIC:
    We have to check wether d2x(t)/dt2 maintains the SAME sign throughout its range (i.e. if constantly > 0 OR if constantly < 0). If it does, then dx(t)/dt is strictly monotonic, and thus "1-1", and thus INVERTIBLE.

  • @funnymonkeys1865
    @funnymonkeys1865 4 года назад +7

    U have stopped ece branch lecture videos please continue networks and other subjects also sir???

  • @AakankshaJohri
    @AakankshaJohri 5 лет назад +3

    Hi, Could you please explain if the below system is invertible or not . And if it is invertible , what is the inverse system in this case ?
    y[n] = x[n]x[n-1]

    • @apoorvpandey5058
      @apoorvpandey5058 4 года назад +1

      It is non invertible

    • @aniketmehtaiitmandi6278
      @aniketmehtaiitmandi6278 2 года назад

      take x1[n]=0 and x2[n]=delta[n] , for both the input signals the output signal is y[n]= 0 hence two distinct input signals lead to same output signal implies that the signal is non invertible

  • @mathbydcsir5385
    @mathbydcsir5385 7 лет назад +10

    plz answer for HW problem

  • @vaideshshankar9899
    @vaideshshankar9899 6 лет назад +9

    can we use u(-t) as input to hw problem 1 such that its output in u(t^2)

    • @kirankumar2432
      @kirankumar2432 6 лет назад +3

      Vaidesh Iyer here the system is replacing t by t^2 so ur going to get the ans.as u(-t^2)brother...

    • @guddankaur9511
      @guddankaur9511 3 года назад +2

      @@kirankumar2432 cant we see it like system is squaring input
      When i/p is u(-t) then sys will square I/p and make it to u(t²).?

  • @420bruce2
    @420bruce2 2 года назад +4

    Can we use 0&π to prove sine terms as non invertible

  • @Fortunecreator321
    @Fortunecreator321 7 лет назад +22

    how can we gurantee the invertiblity of system by only considering impulse or unit step function?

    • @AJitKumar-vg6no
      @AJitKumar-vg6no 5 лет назад +6

      Bro! To solve such problems you should assume the system to be invertible and then try to find out two inputs for which the output is same if there exist two such inputs then the system is non-invertible. You can apply the same approach to solve system stability problems

    • @sravaniamma123
      @sravaniamma123 2 года назад

      @@AJitKumar-vg6no yr tu que you can get some years ago I love❤❤

  • @kaavyasameerasajja6730
    @kaavyasameerasajja6730 6 лет назад +31

    Both are non invertable
    Please give me the solution

  • @ahmadqayyum7505
    @ahmadqayyum7505 6 лет назад +1

    You told three properties to check if the system is dynamic or not? Can we say that those properties hold true for the invertible system too?
    Like if time scaling or time shifting is happening then the system is invertible?

  • @ujjawal1366
    @ujjawal1366 4 года назад +8

    Q2.Differentiation of any constant is zero....so many to one .. so non-invertible
    Q1. When you put u(t) then you will get invertible

    • @gyatika8902
      @gyatika8902 4 года назад +2

      Q2 we cant take constant as i/p becoz i/p must be function of t as given in question x(t)

    • @ujjawal1366
      @ujjawal1366 4 года назад +3

      @@gyatika8902 ....
      Madam, we can take constant as I/P ,see example 1 of this video....
      He has taken 2, -2 as I/p.
      Thanks for response.

    • @ujjawal1366
      @ujjawal1366 4 года назад +3

      @@gyatika8902
      Mam..
      Constant × t^0 = constant
      So it's function of t.

    • @gyatika8902
      @gyatika8902 4 года назад +3

      @@ujjawal1366
      Thanks to correct my concept 😊👍yes u r right we can put constant and answer is noninvertible..

    • @ujjawal1366
      @ujjawal1366 4 года назад +1

      @@gyatika8902
      Your welcome.

  • @punjeshgoriya1076
    @punjeshgoriya1076 6 лет назад +2

    Hello sir
    Will plz explain it with numerical Value than it is better way to understand for us

  • @adityaK2210
    @adityaK2210 3 года назад +4

    Sir, If in the 1st home work question, we take input as u(t) & u(-t) , will the output be x(t^2) ?, system is non - Invertible?

    • @ClickBait123
      @ClickBait123 3 года назад +2

      yes second is invertible

    • @mayankdewan7793
      @mayankdewan7793 3 года назад +1

      @@ClickBait123 bro from my logic it's non-invertible , because if u put u(t) and u(-t) then answer will be 0 .

    • @aryanupadhyay97
      @aryanupadhyay97 2 года назад +1

      @@mayankdewan7793 bro if you diff unit step funcytion u get impulse function

  • @pratyushkesharwani2179
    @pratyushkesharwani2179 4 года назад +5

    Sir please share the answers in next video

  • @mariathasangowrishankar7788
    @mariathasangowrishankar7788 7 лет назад +3

    please add convoloution, fourier transform

  • @rashmiyadav4804
    @rashmiyadav4804 7 лет назад +10

    sir please give answer of HW problems

  • @sikhamishra6106
    @sikhamishra6106 6 лет назад +3

    1. Put x(t2) =t2 ... (-1)2 =(1)2. Hence many to one
    2. Put x(t)= t. Diff we get y(t)=1. Hence many to one

  • @hg084
    @hg084 3 года назад +6

    both HW problems are non-invertible . in 1st problem if we take t=2 and t=-2 , then input will be x(2) and x(-2) and output for both will be y(t)=x (t^2) i.e x(4), so it is many to one mapping . in 2nd question if we take x(t) = u(t) and x(t)=u(-t) then output for both will be zero , so many to one mapping

    • @atmajyotighosh2049
      @atmajyotighosh2049 2 года назад +8

      first is invertible, since we have to check for x(t^2) and not t itself.

    • @Scientificial
      @Scientificial 5 месяцев назад +1

      @@atmajyotighosh2049 it is non invertible, put x(t) = u(-t) and u(t) both will give u(t^2)

  • @barsilgen120
    @barsilgen120 11 месяцев назад

    Thanks sir

  • @dilan5587
    @dilan5587 2 года назад

    for the first question if we give dirac (t) and -dirac(t) one will become dirac t square and other will be - dirac t square . so if t goes to 2 for both wouldn't to result be 0 because dirac (2)= 0 ? and dirac(4) is also 0 other than t =0 dirac would be 0 right or am i confused?

  • @swarajyalakshmi7186
    @swarajyalakshmi7186 6 лет назад +2

    can we use impulse signal in place of step signal

  • @gkk8409
    @gkk8409 7 лет назад +24

    H. W.
    1.non invertible
    2.non invertible

    • @sunandpal
      @sunandpal 6 лет назад +8

      1.Invertible 2.Non Invertible

    • @8revanthvatturi833
      @8revanthvatturi833 6 лет назад +2

      Learn New
      1 Invertible, 2 non Invertible

    • @mr.unique7689
      @mr.unique7689 6 лет назад +2

      revanth Vatturi... in problem 1 check the value at t=-1, and t=1...you got problem 1 is also non invertible...

    • @asabu80
      @asabu80 6 лет назад +9

      mr. unique but we are actually putting values for x(t) not for 't'. right..?

    • @jancydas5569
      @jancydas5569 6 лет назад +4

      2 pblm ans is invertible

  • @YashSharma-dp7ov
    @YashSharma-dp7ov 5 лет назад +6

    The third question is many to one mapping I guess.
    Because as we put different values of t we get same answer i.e. 1.
    Correct me if I am wrong.

    • @syedinamullahshah6076
      @syedinamullahshah6076 3 года назад +1

      we do not put value of t we put value of xt

    • @aniketmehtaiitmandi6278
      @aniketmehtaiitmandi6278 2 года назад

      you need to first of all understand that a system maps a signal to another signal i.e the domain is a set of signals and the codomain itself is a set of signals, so you need to put different values of input signal to check whether you are getting same output signal , if yes then the system is non invertible if not then you need to check the invertibility.

  • @alkakumari-tl5vz
    @alkakumari-tl5vz 6 лет назад +6

    Both non invertible .. right?

  • @abbaswali361
    @abbaswali361 Год назад

    Can a system be invertible and non at the same time ?

  • @prithvi_krishna
    @prithvi_krishna 6 лет назад +15

    Both r non invertible, bcoz many to one mapping.
    1. Take x(t) = u(t) and u(-t) : output is u(t) for both.
    2. Take x(t) = u(t) and -u(-t) : output is impulse(t) for both.

  • @awaisasghar4712
    @awaisasghar4712 7 лет назад +2

    y(t)=(2+sin⁡t)x(t) is invertible or not? plzz ans it

  • @sanskarmodi5975
    @sanskarmodi5975 5 лет назад +1

    Sir agar y(t)=x(2t) m x(t)=u(t) aur x(t)=u(2t) lenge to non invert aaga

  • @neekuenduku
    @neekuenduku 6 лет назад +3

    1. Non invertible, since for negative and positive of some time 't' we get same value of y
    2. Invertible, since for different values of x we get different values of y

    • @jancydas5569
      @jancydas5569 6 лет назад +1

      We need to put different values of input not for different values of time..so the system is invertible

    • @DeepshikhaKumariBEE
      @DeepshikhaKumariBEE 6 лет назад +2

      2. Non-invertible... x1(t)=mt+c1 and x2=mt+c2.. this implies y1(t)=y2(t)=m

    • @jancydas5569
      @jancydas5569 6 лет назад +1

      @@DeepshikhaKumariBEE yes...if the system is non invertibe..this gives two similar output for two different input...and y1 and y2 are straighe lines..and its passes throught origin so m will be equal

  • @gkk8409
    @gkk8409 7 лет назад +4

    sir plz explain the homework sums

  • @priyanshavashishtha3837
    @priyanshavashishtha3837 6 лет назад +2

    2nd ex. is invertible coz for x(t) = r(t) nd u(t) we gt different o/p. so 1-1 mapping . hence I.V.

    • @AshwaniYadavIIT
      @AshwaniYadavIIT 4 года назад

      No
      take x(t) = 1, y(t) = d/dt(1) = 0
      take x(t) = 2, y(t) = d/dt(2) = 0
      Hence non-invertible

  • @satyamverma6029
    @satyamverma6029 3 года назад +16

    1).Invertible 2). Non-invertible

  • @cd-zw2tt
    @cd-zw2tt 2 года назад

    How is the last example a proof of invertibility? How can you show only two examples of different inputs giving different outputs? Of course it is invertible, but isn't that not sufficient proof?

  • @nainamir5654
    @nainamir5654 5 лет назад

    Invertible is first 2nd may or may not be invertible

  • @swarajyalakshmi7186
    @swarajyalakshmi7186 6 лет назад +2

    wt abt the system when y(t)=x(t+1)

  • @rahulkumarsaw1632
    @rahulkumarsaw1632 3 года назад

    draw graph and check , both question is non-invertible

  • @iq_pi7211
    @iq_pi7211 5 лет назад +1

    Delta(t) =1 at t=0

  • @ravindersinghdhillon3942
    @ravindersinghdhillon3942 6 лет назад +2

    1 Non invertible & 2.Invertible

  • @suneethaas7947
    @suneethaas7947 3 года назад +4

    1.invertible
    2.non invertible
    For the first one take u(t) & -u(t) as input will get one to one mapping
    For the second case take input 1 & 2 will get zero because Differentiation of any contast is zero so will get many to one mapping hence its non-invertible system

    • @makindeolalekanmonsuru5349
      @makindeolalekanmonsuru5349 3 года назад

      Why don’t you use a u(t) for the second assignment

    • @santhoshchandchelli6581
      @santhoshchandchelli6581 3 года назад

      @@makindeolalekanmonsuru5349 yeah we can use it.....But make sure that if u have another possibility of inputs that proves a particular maping was wrong.

    • @Vishwa2757
      @Vishwa2757 2 года назад +1

      @@santhoshchandchelli6581 exactly! So for 1st Question : for x(-2) ---> y(-2) = x(4) and x(2)----> y(2) = x(4).Many to one : Non-Invertible.

  • @amanpatel1518
    @amanpatel1518 5 лет назад +1

    sir if i take u(t) and u(2t) as two inputs in 3rd problem the system will not be invertible?

  • @vandanakhairnar5151
    @vandanakhairnar5151 7 лет назад +1

    y(t) = x(t-5) - x(3-t)
    What about this...invertible or not ???

    • @Fortunecreator321
      @Fortunecreator321 7 лет назад +4

      I think it is non invertible as in case if we assume x(t) to be constant,then in that case time shifting does not alter the value of x(t).thus y(t) remain constant for different values of x(t)

    • @vandanakhairnar5151
      @vandanakhairnar5151 6 лет назад

      adarsh raj yeah....but given answer is Invertible..

    • @VivekKumar-zw1wx
      @VivekKumar-zw1wx 6 лет назад +1

      yes it is invertible system

    • @vinaypant557
      @vinaypant557 6 лет назад +1

      What so if we take x(t) =, u(t) and u(-t), then it will be invertible

  • @SaikatPodder
    @SaikatPodder 6 лет назад +1

    Both r invertible ?........reply

    • @reshmipriya3358
      @reshmipriya3358 4 года назад

      Yes..

    • @AshwaniYadavIIT
      @AshwaniYadavIIT 4 года назад

      @@reshmipriya3358 No
      take x(t) = 1, y(t) = d/dt(1) = 0
      take x(t) = 2, y(t) = d/dt(2) = 0
      Hence non invertible

  • @ritvikshekhar4900
    @ritvikshekhar4900 Месяц назад +1

    Both should be non invertible

  • @leolorenzo52raylewis
    @leolorenzo52raylewis 5 лет назад +1

    your explanations are the greatest!! I even let the adds run for you! hahah

  • @RajeshKumarMinaBEE
    @RajeshKumarMinaBEE 2 года назад +2

    1. Invertible
    2. Non-Invertible

    • @mr.infrasonic4084
      @mr.infrasonic4084 10 месяцев назад

      1) for 2 and (-2) both you will get 4 are output... many to one mapping so it is non invertible

  • @shraddhaagrawal7089
    @shraddhaagrawal7089 6 лет назад +4

    Both are non invertible

  • @jiteshbilunia6732
    @jiteshbilunia6732 5 лет назад +6

    1.Non invertible
    2.Non-Invertible

  • @aram8832
    @aram8832 4 года назад +4

    Both seems invertible

  • @ecoboy7829
    @ecoboy7829 2 года назад

    #Neso Academy

  • @ranukaranasinghe3885
    @ranukaranasinghe3885 Год назад

    both are NI easy.

  • @thirupathammakola5571
    @thirupathammakola5571 4 года назад +1

    Both are non invertable

  • @kshtriyesingh5488
    @kshtriyesingh5488 3 года назад

    Both noninvertible

  • @bayzidhossain7422
    @bayzidhossain7422 Год назад

    Non -invertible system and invertible system

  • @saiprasanthkothuri6852
    @saiprasanthkothuri6852 2 года назад

    1) Inv
    2) Non Inv

  • @MOHITSHARMA-hs1qk
    @MOHITSHARMA-hs1qk 6 лет назад +6

    Both are Invertible

  • @rihanrifas905
    @rihanrifas905 5 лет назад +1

    Both are invertible

  • @mretyo
    @mretyo Год назад

    i think 1st one is invertible but 2nd is non invertible

  • @osamamohammed838
    @osamamohammed838 4 года назад +1

    solve plz y(t )= ∫ 𝒙(𝟓𝝉)𝒅𝝉 𝟒
    interate from -infinity to 4 is invertable or not??

    • @apoorvpandey5058
      @apoorvpandey5058 4 года назад +1

      It is invertible bro

    • @ujjawalvats5169
      @ujjawalvats5169 4 года назад +1

      Y(t) will always be constant so it will be many one and non invertible

    • @apoorvpandey5058
      @apoorvpandey5058 4 года назад +2

      @@ujjawalvats5169 no bro it will be always a constant value that i accept but for every different x(t) u will get different y(t) so one-one mapping and hence it will be invertible.
      Just put u(t) first then u(-t) u will get different constant values

    • @ujjawalvats5169
      @ujjawalvats5169 4 года назад

      @Apoorv gaming for every x(t), y(t) will be independent of x(t). No matter what will be x(t). It is just like y(t) =4

    • @apoorvpandey5058
      @apoorvpandey5058 4 года назад

      @@ujjawalvats5169 y(t) is equal to integration of x(5t) so how it will be independent of x(t). Bro just clarify this

  • @SAS_alarms
    @SAS_alarms 6 лет назад +2

    Both the problems having the time invertible property.

  • @rajankarn90
    @rajankarn90 7 лет назад +1

    both invertible .

  • @arti7027
    @arti7027 3 года назад

    1.non invertible
    2.invertible

  • @awesomearchit111
    @awesomearchit111 6 лет назад +3

    Both are non invertible
    Put x(t)=u(-t) and u(t) in first case
    Put x(t)= 2 and 1 in second case

    • @raghunandan2470
      @raghunandan2470 6 лет назад +1

      i think you shouldnt substitute for t but substitute for x(t)

    • @Mangakan_desu
      @Mangakan_desu 5 лет назад

      @@raghunandan2470 same thing

  • @vandanakhairnar5151
    @vandanakhairnar5151 7 лет назад +1

    Both invertible...

  • @411-tulasirammudavathu5
    @411-tulasirammudavathu5 3 года назад

    Non invertible
    Invertiable

  • @sureshreddymaligi4206
    @sureshreddymaligi4206 6 лет назад +1

    1.Invertible,2.Non Invertible

  • @tammalideepak1168
    @tammalideepak1168 5 лет назад +1

    Non Invertible both of them.

  • @kishanbaranwal8331
    @kishanbaranwal8331 4 года назад

    both are non invertible

  • @subha_nik
    @subha_nik 3 года назад

    invertible
    non invertible

  • @mretyo
    @mretyo Год назад

    if possible give ans to me .

  • @wilsonantoney9423
    @wilsonantoney9423 6 лет назад +2

    Both are non - invertible

  • @eddyr978
    @eddyr978 5 лет назад +1

    @6:29 smj nii aaya sir kuch bhi

  • @karthikvenkata6484
    @karthikvenkata6484 3 года назад +1

    1)non-invertible
    2)non-invertible

  • @Fantasticsman
    @Fantasticsman Год назад

    1) NI
    2) NI

  • @prabhakardas4261
    @prabhakardas4261 7 лет назад +2

    1.non invertible,2.invertible

    • @ashokvardineni2872
      @ashokvardineni2872 7 лет назад

      How it is

    • @yourstruly8654
      @yourstruly8654 7 лет назад +4

      +Ashok Vardineni because differentiation of any constant is zero... implies many to one function

    • @evahumaira1213
      @evahumaira1213 7 лет назад +3

      Ashok Vardineni that's why the 2nd one should be non invertible as many to one ...

    • @prithvi_krishna
      @prithvi_krishna 7 лет назад

      Eva Humaira : how is the first one non invertible

    • @tothepoint9914
      @tothepoint9914 7 лет назад +1

      prithvi krishna..... i think it is invertible because when we put u(t) and -u(t) it will give the result u(t^2) and -u(t^2) respectively it is just like a time scaling as previous ex. In this presentation

  • @sunandpal
    @sunandpal 6 лет назад +3

    1.Invertible 2.Non Invertible

  • @chidanandadatta4695
    @chidanandadatta4695 4 года назад +1

    Both are non invertible

  • @aliarman9473
    @aliarman9473 3 года назад

    both are invertible

  • @sureshreddymaligi4206
    @sureshreddymaligi4206 6 лет назад +1

    1.Invertible,2.Non Invertible

  • @princeraazworld7495
    @princeraazworld7495 6 лет назад +6

    1.invertible
    2.non invertible

  • @sairaja6849
    @sairaja6849 6 лет назад +3

    1.Non invertible.
    2.invertible

  • @programmer9884
    @programmer9884 6 лет назад +4

    1. Non-Invertible
    2. Invertible

    • @ThePriyeshpandey
      @ThePriyeshpandey 6 лет назад +1

      let input be x(t)+c where c is a constant and x(t) can be any input, d/dt (input) = x'(t); you have your system performing many to one operation. Derivative operator is non invertible

    • @amansajwan6411
      @amansajwan6411 6 лет назад +5

      Lets put x(t) =u(t) so d/dt u(t) will give unit impulse function and if x(t) is ramp it will give step function si it is a invertible function

    • @gnquince
      @gnquince 5 лет назад

      Put DC value and derivative of DC 0 and for also zero value output also be zero so non invertible

  • @shaillybhai007
    @shaillybhai007 3 года назад

    both are non invertable

  • @guru8353
    @guru8353 7 лет назад

    1.Non invertible
    2.Invertible

  • @Baby12875
    @Baby12875 4 года назад

    1.invertible
    2.non invertible